PT 29 Section 4 #17 Forum

Prepare for the LSAT or discuss it with others in this forum.
Post Reply
bkred

New
Posts: 27
Joined: Mon Feb 22, 2010 1:44 am

PT 29 Section 4 #17

Post by bkred » Tue Aug 24, 2010 9:39 pm

Here's the argument: There is little correlation between the accuracy of a witness's account and the confidence the witness has in the account. And the conclusion is that police officers are advised to disallow suspect lineups in which witnesses can hear one another identifying suspects.

The correct answer is (A), which I don't get. (A) says that confidence is affected, but so what? Confidence has no effect on the accuracy of the account, so why should the police care at all about the confidence levels of the witnesses going up or down? Isn't it accuracy they care about? I realize the other answers aren't any good either, but to me, this one was almost the least likely, along with (D).



Thank you.

JJDancer

Gold
Posts: 1564
Joined: Sun Jul 26, 2009 7:41 pm

Re: PT 29 Section 4 #17

Post by JJDancer » Tue Aug 24, 2010 10:57 pm

I agree that B-E aren't good answers either.
I wish I could help you but I am confused about A as well.
The premise says that there is little correlation between accuracy and confidence. They explicitly say that something can increase/decrease confidence without affecting accuracy. So why would police be advised to not allow lineups where witnesses can hear each other?

It asks the principle underlying this advice - which I guess is that they don't want confidence affected...
But yeah that shouldn't matter.

Sorry I didn't actually help! Maybe someone else can shed some light.

User avatar
Hannibal

Gold
Posts: 2211
Joined: Mon Jul 12, 2010 12:00 pm

Re: PT 29 Section 4 #17

Post by Hannibal » Tue Aug 24, 2010 11:01 pm

Little correlation is not no correlation, perhaps?

Edit: wait, they would try to avoid factors that increase or decrease confidence without affecting accuracy so they can better judge the truth behind the confidence of each witness.

JacobH

New
Posts: 38
Joined: Sun Feb 28, 2010 10:04 pm

Re: PT 29 Section 4 #17

Post by JacobH » Tue Aug 24, 2010 11:32 pm

Lets think about this for a second.

Conclusion states that POs are advised to disallow any instance in which witnesses can interact with each other in choosing out of a lineup.

The question asks us to find the principle that allows for the advice given to the POs to be valid.

If we leave accuracy out of the equation here, because that is not what is asked of us, then we can assume that the POs care about the witness's confidence. What for we can't infer from the passage.

This leaves us with only answer A, which correctly connects the conclusion to the stimulus right before it.

User avatar
Anaconda

Silver
Posts: 605
Joined: Sun Jun 06, 2010 3:51 pm

Re: PT 29 Section 4 #17

Post by Anaconda » Wed Aug 25, 2010 12:27 am

I kind of feel this would be better as an assumption question. A pretty much sums up the principle of the last sentence, with a tie-in from the first sentence.

Kaplans says this is a one-star question - I think it's a bit tougher than that.

Want to continue reading?

Register now to search topics and post comments!

Absolutely FREE!


Post Reply

Return to “LSAT Prep and Discussion Forum”